Question:

Make the correct pair-

disease with its affected organ

Choose the correct answer from the options given below-

Show Hint

For questions involving diseases and their affected organs, it is helpful to have a basic understanding of common diseases, their causes (bacterial, viral, deficiency, et(C)), and the primary systems or organs they target. This knowledge is fundamental in general science and biology.
Updated On: May 22, 2025
  • I-a, II-b, III-c, IV-d
  • I-d, II-a, III-b, IV-c
  • I-b, II-c, III-d, IV-a
  • I-c, II-d, III-a, IV-b
Hide Solution
collegedunia
Verified By Collegedunia

The Correct Option is B

Solution and Explanation

Step 1: Understand the objective.
The question requires matching specific diseases (Column I) with the primary organs or systems they affect (Column II). 
Step 2: Match each disease with its affected organ/system.
(I) Diphtheria: This is a bacterial infection primarily affecting the throat and respiratory tract, leading to breathing difficulties and other symptoms. So, (I) matches with (d) Throat and Respiratory tract.
(II) Jaundice: Jaundice is characterized by yellowing of the skin and eyes due to an excess of bilirubin, which is typically a sign of an underlying issue with the liver or bile ducts. So, (II) matches with (a) Liver.
(III) Rickets: Rickets is a condition in children that causes soft and weak bones, primarily due to a severe deficiency of Vitamin (D) So, (III) matches with (b) Bones.
(IV) Tetanus: Tetanus is a bacterial infection that affects the nervous system, leading to painful muscle spasms and lockjaw. The toxins produced by the bacteria interfere with nerve signals to the muscles. So, (IV) matches with (c) Nervous system and muscles. 
Step 3: Compile the correct matching pairs.
The correct matches are:
(I) - (d)
(II) - (a)
(III) - (b)
(IV) - (c)
This sequence corresponds to I-d, II-a, III-b, IV-(C) 
Step 4: Compare the compiled sequence with the given options.
Option A: I-a, II-b, III-c, IV-d (Incorrect)
Option B: I-d, II-a, III-b, IV-c (Matches the derived sequence)
Option C: I-b, II-c, III-d, IV-a (Incorrect)
Option D: I-c, II-d, III-a, IV-b (Incorrect)
The final answer is $\boxed{\text{I-d, II-a, III-b, IV-c}}$.

Was this answer helpful?
0
0